LSAT and Law School Admissions Forum

Get expert LSAT preparation and law school admissions advice from PowerScore Test Preparation.

User avatar
 Dave Killoran
PowerScore Staff
  • PowerScore Staff
  • Posts: 5853
  • Joined: Mar 25, 2011
|
#41141
Complete Question Explanation
(The complete setup for this game can be found here: lsat/viewtopic.php?t=15614)

The correct answer choice is (D)

Since M, G, and I must all have joined the law firm after J, 1965 is the latest year in which J could have joined the law firm. Therefore, answer choice (D) is correct.
 lancasternovoa
  • Posts: 1
  • Joined: Jul 26, 2017
|
#37675
On question 16 why is the answer "D" 1965? Shouldn't it be 1964 if H, could be before,K is before per the rules,N could be that brings J to 1964. Is O also considered that it could be before J?

thanks,
C.
 Eric Ockert
PowerScore Staff
  • PowerScore Staff
  • Posts: 164
  • Joined: Sep 28, 2011
|
#37717
Hey C

When you are asked for the "latest" year that J could be replaced, look to how many variables MUST be after him. You know that James is before Gregg who is before Ivan. Also, James is before MacNeil. Therefore, since there are three people who MUST be after James, he can't be 1968, 1967, or 1966.

We don't know where J is in relation to H, N, or O. So J COULD be after all three of these variables. And we know that J MUST be after K. So with four variables who either CAN (H, N, & O) or MUST (K) be before J, he CAN be as late as 1965.

Hope that helps!

Get the most out of your LSAT Prep Plus subscription.

Analyze and track your performance with our Testing and Analytics Package.